Sei sulla pagina 1di 11

NEW GMAT - VERBAL ABILITY - CRITICAL REASONING CONCLUSION QUESTIONS

Conclusion questions in the Critical Reasoning section ask you to identify the conclusion that can be logically arrived at on the basis of the information in the given passage. The conclusion questions may be worded in many different ways such as: (i) Which of the following conclusions can be properly drawn from the statements above? (ii) The statements above best support the conclusion that .... (iii) If the statements above are true, which of the following must also be true? (iv) From the information above, it can be properly concluded that .... (v) The author of the above passage argues that .... (vi) Which of the following is best supported by the passage above? We shall first give you a number of typical examples of conclusion questions and also analyse them for your benefit to guide you on how such questions should be tackled. Apart from spotting the correct answer, we shall also explain why each of the other choices is wrong. Try to answer these questions by yourself first before reading the analyses below. Example 1 Macklin Countys mandatory driver-training program for fifteen-year-old high school students was discontinued ten years ago, and since then private driver-training program enrollment has increased only slightly. During the same period the countys accident rate increased by 12 percent for drivers between the ages of sixteen and twenty-five, even though traffic conditions changed little. Which of the following conclusions is best supported by the information above? (A) Graduates of public driver-training programs tend to be safer drivers than graduates of private drivertraining programs tend to be. (B) Drivers between the ages of sixteen and twentyfive, on average, have higher accident rates than do drivers over the age of twenty-five. (C) The majority of people who take some type of driver-training program do not become involved in an accident when they are between the ages of sixteen and twenty-five (D) Changes in traffic conditions affect the accident rate of drivers over the age of twenty-five more than they do that of drivers between the ages of sixteen and twentyfive (E) As a group, drivers between the ages of sixteen and twenty-five become involved in fewer accidents if they participate in a driver-training program than if they do not. Analysis (Mandatory means legally required or compulsory) The first part of the first sentence gives the information that the Macklin Countys compulsory driver-training program for 15-year old students was discontinued ten years ago. The second part adds that, since then, enrollment in the (voluntary) private driver-training program in the County has shown only a slight increase. The second sentence says that, during the last ten years (after the discontinuance of the compulsory requirement), the Countys accident rate for drivers between the ages of 16 and 25 increased by 12%, even though traffic conditions had changed little. We have been asked to identify the choice that forms the conclusion best supported by the given information. The obvious conclusion that can be drawn from the given information is that, if there is a compulsory drivertraining program for 15 year old students, drivers between the ages of 16 and 25 get involved in fewer accidents. In other words, drivers between the ages of 16 and 25 who had undergone a driver-training program get involved in fewer accidents than those who had not undergone such a program. It is (E) which says this, and is the answer. (A) would have been correct if the private drivertraining programs had attracted all the teenagers who would have otherwise been trained by the compulsory public driver training program. But the second part of the first sentence of the narrative specifically states that private driver-training program had attracted very few additional enrollments during this period, implying that many of those who had been involved in subsequent accidents had not undergone any training programs at all. So, (A) is not a conclusion that is warranted by the information in the narrative. The narration compares the accident rates involving only drivers between the ages of t6 and 25 before and after the discontinuance of the mandatory driver-training program for the 15-year old students. It does not compare the accident rates for drivers younger than 25 with the accident rates for drivers over the age of 25. So, (B) is not a conclusion that is warranted by the information in the narrative. The passage merely implies that youngsters who had undergone a driver-training program are less likely to get involved in accidents than are those who have not undergone such a program. Choice (C) which states that a majority of youngsters who take some type of drivertraining program do not get involved in accidents at all goes much beyond the scope of what is stated in the passage, and is wrong. The narration specifically says that there had been little change in the traffic conditions during the period covered by the survey. (D), which refers to the contribution of changes in traffic conditions to the accident rates, is irrelevant to the information in the given passage, and is not the answer. (Note that, in this example, (A), (B) and (C) are wrong because they go beyond the scope of what is stated in the passage, while (D) is wrong because it is irrelevant to the information in the passage.) Example 2 A group of subjects saw a film of two cars colliding. Immediately afterward, half of the group were asked a set of questions about the cars bumping into one another, while the other half were asked the same questions, but with the verb smash substituted for bump. In later descriptions of the filmed collision, subjects in the latter half were more likely to remember seeing broken glass.

New GRE - Logical Reasoning The experiment described above best supports which of the following conclusions about eye-witness testimony? (A) A witness who is agitated at the time of an event is likely to give less accurate testimony than is a calm witness. (B) A witnesss perception of an event will be distorted if inflammatory language is used by the questioner. (C) The manner in which a witness is questioned after an event can influence the recollection of the witness. (D) Most eyewitness testimony can be assumed to contain inaccurate elements. (E) Special questioning techniques can be developed that will be more conducive to eliciting accurate information from witnesses than traditional techniques have been. Analysis (In this question, the word subjects in the first line means persons) What the narrative states is that a number of persons were first shown a film depicting the collision of two cars. Subsequently, they were divided into two groups, and those in the first group were given a questionnaire in which the word bump was used, while those in the second group were given the same questionnaire with a single variation - the word bump having been substituted by the word smash. The narrative then states that the persons in the second group, for whom the word smash was suggested, were more likely to remember seeing glass having been broken during the collision. What this means is that though two witnesses had seen the same event, their recollection of that event can be influenced by the manner in which questions are put to them. So, (C) is the answer. (A) talks of two different witnesses one of whom was agitated at the time of seeing the event while the other was calm. But the passage talks of a single group of persons who were all shown the same film of two cars colliding. This single group was only subsequently divided into two groups just for the sake of being given two different questionnaires containing just one significant variation. There is no evidence in the passage that the first of these groups had been agitated even while seeing the film while the other had remained calm. So, (A) is irrelevant to the information in the passage. Even assuming that the word smash is more inflammatory than the word bump, the result of the experiment described in the passage is that those to whom the word smash was suggested were more likely to remember seeing broken glass, implying that they would be more accurate than the other group for whom the word bump had been suggested. So, (B) contradicts what is stated in the passage, and is not the answer. (If the last sentence in the passage were, In later descriptions of the filmed collision, subjects in the latter half were more likely to imagine seeing broken glass, this choice would have been the correct answer.) The statement in (D), Most eyewitness testimony can be assumed to contain inaccurate elements, is a sweeping generalisation which cannot follow as a necessary conclusion from a single experiment that is described in the passage. (All conclusion questions will be based on the narration of a single event having a limited scope, and any such sweeping generalisation in one of the choices is likely to render it wrong.) (E) extends the scope of the result of the experiment much beyond what is specifically stated in it. So, though what is stated in (E) may be true, it does not follow
immediately from the given passage, and is not the answer. (Note that, among the wrong choices in this example, (A) is irrelevant to the information in the passage, (B) contradicts the given information, (D) is an unwarranted and sweeping generalisation and (E) extends the scope of the given information beyond what can be reasonably inferred.) Example 3 In a study of more than 8,000 people using ten beaches on two of the Great Lakes, ecologists from the University of Toronto determined that the rate of respiratory and gastrointestinal illness among people who had been swimming was 69.6 per 1000, whereas the respiratory and gastrointestinal illness rate among those who had not been swimming was only 29.5 per 1,000. Which of the following conclusions can be most properly drawn from the data above? (A) People tend to underestimate the risks of swimming in these lakes. (B) Respiratory and gastrointestinal illnesses occur at a higher rate as a result of swimming in either of these lakes than they do as a result of swimming in any other lake. (C) Illnesses of kinds other than respiratory and gastrointesinal are not likely to be associated with swimming in either of these two lakes. (D) The association between swimming in these lakes and respiratory and gastrointestinal illness is some evidence for a causal relationship between them. (E) A large percentage of the people who swim in these lakes are immune to the diseases that swimming may cause. Analysis (The Great Lakes of USA are Lake Superior, Lake Michigan and Lake Huron.) The passage says that a sample study of 8,000 persons using ten beaches of two of the Great Lakes showed that nearly 7% of the swimmers among them had respiratory and gastrointestinal illnesses, while only 3% of the non-swimmers among them had these illnesses. We have been asked to spot that choice which can be a logical conclusion that can be validly drawn from this study. From the given information, you may be tempted to conclude that swimming in these lakes can cause respiratory and intestinal illnesses. (You cannot conclude that swimming in these lakes will cause these illnesses because, after all, the increase in percentage of swimmers who are affected is a mere 3%, and 93% of the swimmers are found to be not suffering from these diseases.) 1 But you do not know how many beaches there are along the shores of these two Great Lakes, how many thousands of persons are using these beaches and whether 10 beaches and 8,000 people are large enough and representative samples from which a conclusive inference can be drawn about all those who swim in these lakes from all the other beaches along their shores. (For example, if these 10 beaches are located near factories which pollute the water in the region of the lake near them, the phenomenon may be restricted to the water near these beaches alone, and not to the entire waterspread.) Therefore we can, at best, conclude from the given data that there is some evidence of a cause and effect

New GRE - Logical Reasoning


relationship between swimming in the two lakes and the incidence of respiratory and intestinal diseases (implying that a further detailed study may have to be undertaken before such a relationship could be established as true). (D) is what makes such a tentative statement, and is the answer. Even the sample study shows that, between nonswimmers and swimmers in these lakes, the difference in the incidence of respiratory and gastrointestinal illnesses is only 3% and that, even among the swimmers, nearly 93% are not affected by these illnesses. Nor does the passage indicate that these illnesses are serious in nature, as otherwise the affected persons will not be continuing with their swimming. Therefore, at least for a large percentage of the people, swimming in these lakes does not constitute a risk at all , and the question of people underestimating the risk does not arise. So, (A) is a sweeping generalisation not warranted by the information in the passage. The sample study was conducted only among people who were swimming from ten beaches on two of the three Great Lakes. The findings of this study have no relevance to swimmers in other lakes. (B) seeks to extend the scope of what is stated in the passage beyond what immediately follows from it, and is not the answer. The study that is referred to was limited only to two specific illnesses, and cannot throw any light on whether or not any other illness is associated with swimming in these two Great Lakes. (C) also seeks to extend the scope of what is stated in the passage beyond what immediately follows from it, and is not the answer. (E) talks generally of diseases that swimming may cause whereas the study was restricted to only two diseases -respiratory and gastrointestinal. So, (E) is in the nature of a sweeping generalisation which is not warranted by the information in the passage. (Note that, among the wrong choices, two are sweeping and unwarranted generalisations and two try to extend the scope of what is stated in the passage.) You can notice from the examples discussed above that even the wrong choices given underneath a question are not arbitrarily worded, but follow a specific pattern. Therefore, while evaluating each of the answer choices, ask yourself the questions: (i) Does this choice contradict a specific information in the passage? (ii) Does the choice extend the scope of what is stated in the passage beyond what can be immediately inferred? (iii) Does the choice make a sweeping generalisation from a single instance referred to in the passage? (iv) Is the statement in this choice irrelevant to the information in the passage? If the answer to any of these questions is yes, that choice is bound to be a wrong answer, and can be discarded.

We are giving below 20 questions on Conclusions for your practice.


SET 1 1. A package is never accepted for delivery by the delivery service unless it is within the established size limits. All packages accepted for delivery by the delivery service have a return address. If the statements above are true, which of the following must also be true? (A) The delivery service charges more for heavier packages than for lighter packages. (B) The delivery service will always accept for delivery a package that is within the established size limits. (C) If a package is within the established size limits and has a return address, it will be accepted for delivery by the delivery service no matter how heavy the package is. (D) A package that is not within the established size limits but has a return address is never accepted for delivery by the delivery service. (E) The delivery service does not charge for packages that must be returned to the sender. 2. Each increase of 1 percent in real disposable personal income per capita will increase the share of the electorate for an incumbent by about 2.2 percentage points, other things being equal. Since 1952 there has been a decline in real disposable income during only one presidential election year. The incumbent lost that election. Which of the following conclusions can be properly drawn from the statements above? (A) When an incumbent runs for office, he or she is likely to win. (B) Political parties should take care to put forth a candidate who seems prosperous. (C) Presidential candidates should put their greatest efforts into improving their public image. (D) Because a presidential campaign requires the expenditure of large amounts of money, it frequently impoverishes a candidate and his or her supporters. (E) The outcome of a presidential election is substantially affected by factors other than the ideological positions of the candidates. 3. A group of scientists studying sound patterns in certain monkeys found that the adult monkeys had distinct alarm calls for eagles, leopards, and snakes. The infant monkeys, however, used the eagle alarm for any flying animal, the leopard alarm for any walking animal, and the snake alarm for any long, thin animal. Which of the following is the most logically defensible explanation of the monkeys behavior? (A) Neither the adult monkeys nor the infant monkeys observed in the experiment had speech of any sort (B) The infant monkeys had yet to learn to distinguish one kind of animal from another within the three groups (C) The infant monkeys mimicked the adult monkeys with no understanding of the significance of the three different sounds (D) The infant monkeys had not yet learned to make the three alarm calls as well as the adult monkeys could (E) Only the adult monkeys recognized that eagles, leopards, and snakes posed dangers to them 4. Calcium consumed in recommended levels is stored by bones and so helps increase bone mass, especially for teenagers. Beginning at age 35, the stored calcium is used without being replenished. The more bone mass an individual has, the less likely it is that the individual will develop osteoporosis, a disease that makes bones brittle, especially in people over age 50. Which of the following is a conclusion that can be properly drawn from the statements above?

New GRE - Logical Reasoning


(A) People who steadily increase the amount of calcium they consume from age 35 on are less likely to suffer osteoporosis than are people who do not steadily increase the amount of calcium they consume from age 35 on. (B) People should steadily increase the amount of calcium they consume from ages 35 to 50 to prevent osteoporosis. (C) People who consume the recommended levels of calcium during their teen years are less likely to suffer osteoporosis than are people who increase their consumption of calcium to the recommended levels between the ages of 35 and 50. (D) People who are at least 50 years old and who suffer from osteoporosis can be cured by immediately increasing the amount of calcium they consume. (E) People who are at least 50 years old and who suffer from osteoporosis did not consume calcium when they were teen-agers 5. Clay absorbs radiation with time, releasing it only when heated. By heating a clay sculpture and measuring the radiation it releases, experts can determine to within a century when the sculpture was last heated. The original firing of the finished sculpture might be the occasion of that most recent heating. Experts who obtain the year A.D. 1450 as an estimate for a given sculpture using the method described above would thereby most seriously undermine any claim that the sculpture was made in (A) A.D. 1000 (B) A.D. 1400 (C) A.D. 1450 (D) A.D. 1500 (E) A.D. 1900 6. To be a good debater, one must be intelligent. Some good debaters, however, are also contentious, and contentious persons are always boring Which of the following conclusions can be properly drawn from the statements above? (A) All good debaters are boring (B) All contentious persons are good debaters (C) Only good debaters are contentious (D) Some intelligent persons are boring (E) Most intelligent persons are boring 7. Dryden makes deliveries only if he has his employers permission. Drydens employer permits Dryden to make deliveries alone during day-light hours, but permits him to make deliveries after dark, only if both Stan and Estey are with him. Which of the following conclusions can be logically inferred from the statements above? (A) If it is dark and Estey is with Dryden, then Dryden will make deliveries. (B) If it is dark and Dryden is making deliveries, then Stan is with him. (C) If Stan is not with Dryden, then Dryden is not making deliveries. (D) If Dryden is making deliveries, then Stan and Estey are with him. (E) If Dryden is making deliveries alone, then it is before 6.00 p.m 8. After the national speed limit of 55 miles per hour was imposed in 1974, the number of deaths per mile driven on a highway fell abruptly as a result. Since then, however, the average speed of vehicles on highways has risen, but the number of deaths per mile driven on a highway has continued to fall. Which of the following conclusions can be properly drawn from the statements above? (A) The speed limit alone is probably not responsible for the continued reduction in highway deaths in the years after 1974 (B) People have been driving less since 1974 (C) Driver-education courses have been more effective since 1974 in teaching drivers to drive safely (D) In recent years highway patrols have been less effective in catching drivers who speed (E) The change in the speed limit cannot be responsible for the abrupt decline in highway deaths in 1974 9. The countrys continued existence as a nation depends on its taking active measures to preserve national security. Some of the measures may not be morally acceptable, but they must be undertaken, nevertheless. If the statements above are true, it follows that (A) It is impossible to act morally in foreign policy. (B) Each country is morally obliged to defend its own security. (C) Active measures to preserve national security must be immoral. (D) The pursuit of continued national existence is morally justifiable. (E) Adherence to morality is less important than ensuring the countrys security. 10. For some women the cost of giving birth can be an unexpectedly large burden. The average normal birth now costs $3,200, and a birth with complications can cost thousands of dollars more. Of women in the primary childbearing age range of eighteen to twentyfour, who account for about 40 percent of all births in this country annually, more than 25% have no health-care insurance to pay maternity costs. If the statements above are true, which of the following must also be true? (A) Each year, about 75 percent of all births in this country are to women who have health-care coverage of maternity costs. (B) Each year, about 60 percent of all births in this country are to women who are younger than eighteen or older than twenty-four. (C) For an average birth, health-care insurance pays about 75% of $3,200. (D) In this country, about 75 percent of the women who do not have health-care coverage of maternity costs are younger than eighteen or older than twentyfour. (E) In this country, nearly 75 percent of the women in the primary childbearing age give birth with no complications. SET 2 1. Women generally do not receive lower pay for doing precisely the same work as men. Rather, regardless of their skills, women have disproportionately high representation in jobs traditionally paying lower wages. If jobs paid according to the skills and education required by them, this de facto segregation would not exist. If the statements above are true, which of the following is most likely to be true? (A) Some jobs performed by more men than women pay higher wages than jobs requiring comparable skills, but performed by more women than men

New GRE - Logical Reasoning


(B) Regardless of sex or occupation, a person should be paid an amount sufficient to support his or her household (C) As men enter a field, earnings tend to rise, not only for men but also for the women in the field (D) Little percentage gain has occurred in the number of women holding jobs traditionally held by men (E) Generally accepted methods are available for evaluating the level of skill required in, and hence the appropriate pay for, any job traditionally held by women 2. Marijuana has been blamed for causing apathy, loss of motivation, narrowing of interests, and similar signs of depression in people who use it heavily. There is no evidence that the drug does indeed produce such an a motivational syndrome. On the contrary, young people who are depressed and have low expectations for themselves - and whose parents expect little of them are more likely than others to make heavy use of marijuana. Which of the following would provide the most logical concluding sentence for the paragraph above? (A) Therefore, it can be concluded that marijuana does not produce an amotivational syndrome (B) Hence, it can be concluded that the evidence concerning the amotivational syndrome is at best inconclusive (C) In fact, their parents low expectations for them often are a major factor inducing young people to use marijuana heavily (D) In other words, heavy use may be a symptom of depression and low self-esteem rather than a cause (E) Moreover, some studies indicate that heavy users of marijuana experience a broadening of interests and a gain of motivation 3. The university constitution defines the powers of two governing bodies. The general council, which currently retains authority over constitutional matters, consists of all the universitys living alumni. The twenty-member senate decides routine matters by majority vote. A new proposal suggests making a unanimous vote in the senate sufficient to change the constitution. If the statements above are true, which of the following must follow on the basis of them? (A) The proposal will take effect only if it receives the senates unanimous support. (B) Currently each member of the senate has power of veto over any proposal that comes before that body. (C) Adopting the proposal would allow passage of constitutional changes without the general councils approval. (D) Any of the universitys alumni can fill any one of the twenty seats on the senate. (E) Adapting the new proposal would increase the voice of the alumni in the conduct of university affairs. 4. The greater the division of labor in an economy, the greater the need for coordination. This is because increased division of labor entails a larger number of specialized producers, which results in a greater burden on managers and, potentially, in a greater number of disruptions of supply and production. There is always more division of labor in market economies than in planned economies. If all of the statements above are true, then which of the following must also be true? (A) Disruptions of supply and production are more frequent in planned economies than in market economies. (B) There are more specialized producers in planned economies than in market economies. (C) The need for coordination in market economies is greater than in planned economies. (D) A managers task is easier in a market economy than in a planned economy. (E) Division of labor functions more effectively in market economies than in planned economies. 5. Investigators studying the facial expressions of children who are born both blind and deaf have found that certain patterns of muscular movement, such as those involved in smiling, are similar in form to the patterns of muscular movement in the expressions shown by children without these handicaps. Which of the following conclusions can most reliably be drawn from the above information? (A) Facial expressions are not part of the language of gesture that is taught by each society to its children (B) The facial expressions found to be similar are the same in human beings everywhere. (C) The facial expressions found to be similar are not always learned by imitation of sight or sound. (D) Facial expressions denoting positive emotions, such as smiling, are more easily learned than those denoting negative emotions. (E) The children who smiled were classifiable as normal for educational purposes. 6. Joe: All Americans like ice-cream. James: No, that is not true, I know a couple of Americans who just love chocolates. From the above conversation, we can say that James has misunderstood Joe to mean (A) Only Americans like ice-cream (B) Joe does not like ice-cream (C) Americans like only ice-cream (D) America does not produce chocolates (E) Those who like ice-creams will not like chocolates 7. If a car was built after 1965, it has harness-style seat belts in the front seats. From which of the following choices does the statement above logically follow? (A) Harness-style seat belts first appeared in cars in the late 1950s. (B) No seat belts other than harness-style seat belts were used in front seats of cars after 1965, but, all cars built after 1965 were required to have seat belts in the front seats. (C) Some cars built before 1965 have harness-style seat belts in the front seats. (D) Seat belts were not required in cars built before 1965, but many people who had observed the benefits of seat belts on racing cars had them installed. (E) All cars built after 1960 were required to have seat belts in the front seats, but not necessarily harnessstyle seat belts. 8. Most canvases used by artists in the pre-industrial United States were imported from Europe, but the wooden stretchers on which the canvas was mounted were always made from American wood. Retailers of art supplies, both in the United States and Europe, often stamped blank canvases with their names and addresses.

New GRE - Logical Reasoning


Completed paintings brought from Europe to America were frequently taken off their European stretchers for shipping and remounted on American stretchers after their arrival. A cultural historian could legitimately use the information in the passage above in establishing that a painting of that era done on (A) canvas sold by a European retailer and mounted on a stretcher of American wood is probably an American painting (B) canvas sold by an American retailer and mounted on a stretcher of American wood is probably an American painting (C) canvas sold by an American retailer and not mounted on a stretcher is probably not an American painting (D) unmarked canvas mounted on a stretcher of American wood is probably an American painting (E) unmarked canvas that is not mounted on a stretcher is probably not an American painting 9. For a local government to outlaw all strikes by its workers is a costly mistake, because all its labor disputes must then be settled by binding arbitration, without any negotiated public-sector labor settlements guiding the arbitrators. Strikes should be outlawed only for categories of public-sector workers for whose services no acceptable substitute exists. The statements above best support which of the following conclusions? (A) Where public-service workers are permitted to strike, contract negotiations with those workers are typically settled without a strike. (B) Where strikes by all categories of public-sector workers are outlawed, no acceptable substitutes for the services provided by any of those workers are available. (C) Binding arbitration tends to be more advantageous for public-service workers where it is the only available means of settling labor disputes with such workers. (D) Most categories of public-sector workers have no counterparts in the private sector. (E) A strike by workers in a local government is unlikely to be settled without help from an arbitrator. 10. If it is true that the streets and the sidewalks are wet whenever it is raining, which of the following must also be true? Consider each choice independently and choose all those that apply. (A) If the streets and sidewalks are wet, it is raining. (B) If the streets are wet but the sidewalks are not wet, it is not raining. (C) If it is not raining, the streets and sidewalks are not wet.

New GRE - Logical Reasoning

ANALYSIS
SET 1 Answers 1.D 2.E 3.B 4.C 5.E 6.D 7.B 8.A 9.E 10.B Analysis 1. The given passage states that, for being accepted by the delivery service, a package must fulfill both of the two conditions: (i) it must be within the established size limits; and (ii) it must contain a return address. We have been asked to spot that choice which is a logical conclusion that can be drawn from these two statements. The given passage does not refer to the charges of delivery at all. So, (A) extends the scope of what is stated in the passage, and is not a necessary conclusion from it. (B) contradicts the second condition that, for being accepted, the package must necessarily have a return address also. (C) looks like a correct answer, but is not valid because, the statement that the delivery service will never accept packages which do not fulfill the two conditions does not imply that it will necessarily accept all packages which fulfill these two conditions. (For example, it may yet reject a package for other reasons such as it being packed loosely, or that it contains a highly inflammable substance, a poisonous chemical etc.). This choice also extends the scope of the given information. (D) reiterates that a package which conforms to one of the rules but not the other will not be accepted by the delivery service, and is the answer. As pointed out with reference to (A), the passage does not talk about charges at all. Therefore, (E) also extends the scope of the given information, and is not a necessary conclusion from it. 2. (The incumbent means a person who is holding office as President of USA at the time of the election, and is seeking a second-term.) What the passage says is that if, during a Presidents term, there had been an increase in the per-capita income of the people, his chances of reelection would be brighter. It even quantifies it by stating that, for every 1% increase in the per-capita income, he can get 2.2% more votes than he would otherwise have got. It seeks to substantiate this argument by pointing out that, in the only election year since 1952 when the per-capita income had declined, the incumbent President was defeated in the election. We have been asked to spot that statement among the choices which would logically follow from this passage. (A) contradicts the specific instance cited in the passage of an incumbent President having lost an election, and is wrong. (B) and (D) are irrelevant to the issue, because the passage talks of the prosperity of the general public, and not of the candidate. (C) is also irrelevant to the passage because the passage does not talk of the public image of the incumbent at all. (E) is what can be directly inferred from the passage, because it states that the result of an election is not solely determined by the ideological positions of the candidates, but can be influenced by other factors such as an increase in the per-capita income of the people. So, (E) is the answer. 3. What the passage means is that, among monkeys, there are different kinds of alarm signals for different birds, for different walking animals and for different reptiles. Adult monkeys give many different alarm calls depending on which enemy bird, or which enemy animal, or which enemy reptile is on the prowl. But infant monkeys know only three alarm calls the first for any flying bird, the second for any walking animal, and the third for any reptile. The obvious explanation for this behaviour of infant monkeys is that they first learn only to distinguish among the three broad groups -birds, animals and reptiles. It is only when they mature into adults that they are able to make distinctions among different birds, different animals and different reptiles. So, (B) is the answer. (A) contradicts the specific information in the passage that both adult and infant monkeys do use sound patterns that communicate information to the other monkeys in the group. (C) is incorrect because, in the case of both adult and infant monkeys, the word alarm is used. We can therefore infer that infant monkeys also know that the significance of producing the sound is to alert other monkeys of impending danger, or to seek help. This choice therefore contradicts the information in the passage, and is wrong. (D) does not follow from the given passage, since the passage does not mention the relative skills with which the adult and the infant monkeys give the three basic alarm calls that relate to a flying animal, a walking animal and a long, thin animal. (D) therefore seeks to extend the scope of the information in the given passage, and is wrong. (E) is wrong for the same reason as (C), since the word alarm is used in the case of infant monkeys also, implying that infant monkeys also recognize that eagles, leopards and snakes posed dangers to them. Thus (E) also contradicts the information given in the passage. 4. The first sentence says that, if calcium is taken in recommended doses by teenagers, it gets stored by their bones. The second sentence says that, beyond the age of 35, it is not possible to replenish (meaning to add to the stock of) calcium in the body, and the bodys needs for calcium beyond this age are met from the stock already available in the bones. The third sentence says that those who have more bone mass in their system are less likely to suffer from osteoporosis in their old age beyond 50. The obvious conclusion from these statements is that, if a person wishes to avoid osteoporosis in old age, he should start consuming the recommended levels of calcium when he is in his teens, and continue to take them. Neglecting such intake when one is in his teens, but consuming calcium after one crosses 35 (when the bodys capacity to absorb calcium has ceased), will not help him to avoid osteoporosis in his old age. In other words, this means that, between a person who had consumed sufficient quantities of calcium in his teens, and another person who started consuming calcium after he had crossed 35, the former is more likely to escape from osteoporosis in old age than the latter. It is (C) which states this, and is the answer. Since the passage says that calcium in the bone is not replenished beyond the age of 35, consumption of calcium after the age of 35 does not result in the prevention of

New GRE - Logical Reasoning


osteoporosis in old age. So, (A), (B) and (D) contradict the specific information given in the passage, and are wrong. What the first sentence implies is that a person who had not consumed calcium in recommended levels when he was a teenager is likely to develop osteoporosis after the age of 50. (E) makes a sweeping generalization that persons who develop osteoporosis after the age of 50 had not consumed any calcium at all when they were teenagers, and is wrong. 5. A clay sculpture requires to be heated for being preserved forever in the future. The given experiment explains a way to determine the date (within a century) when a particular sculpture that is available today was last heated. The question, on the other hand, asks us to identify that year among the choices in which the sculpture could not have been made. The second para implies that experts have obtained 1450 as an estimate of the year in which the sculpture was last heated. Since it is given that the method of dating is accurate within a century, the date when the sculpture was heated could well be 1400, 1450 or 1500. So, (B), (C) and (D) are all possible dates when the sculpture was made and also heated for the last time. It is also possible that the sculpture was made in 1000, but was again heated between 1400 and 1500. So, (A) is a possible claim. If the sculpture had been made only in 1900, it could not have been last heated in 1450 or thereabouts. So, it is (E) which is not possible, and is the answer. 6. (Contentious means quarrelsome) We have been asked to state which conclusion can be logically drawn from the given statements. The first statement means that all good debaters are intelligent. The second statement says that some good debaters are contentious. Combining these two, we can conclude that some intelligent persons (who are also good debaters) are contentious. (i) The third statement is that all contentious persons are boring. (ii) When we combine (i) and (ii), we can conclude that some intelligent persons (who are both good debaters and contentious) are boring. Among the choices, it is (D) which states this, and is the answer. 7. (A) is wrong since Dryden cannot make deliveries after dark unless Stan is also with him along with Estey. (B) conforms to the given conditions, and is the answer. (In fact, in addition to Stan, Estey will also be with him in this case). (C) is wrong, since, during daytime, Dryden can make deliveries even when Stan is not with him. (D) is wrong for the same reason. (E) is designed to trip you. You cannot assume that darkness falls always at 6 p.m. In summer, in northern latitudes, there could be daylight even beyond 10 p.m. 8. The first part of the statement says that the imposition of the speed limit of 55 mph in 1974 resulted in an abrupt fall in the accident rate. The second part says that, while the speed limit has not been further reduced, the accident rate has continued

to diminish, though the average speed of vehicles on the highways has risen (but still being less than the speed limit of 55 mph). We have been asked to spot the logical conclusion that can be drawn from these facts. The obvious conclusion from the given statements is that, while the imposition of the speed limit was certainly a cause for reduction in accident rate, it had not been the only cause because, in that case, the continuing fall in the accident rate when the speed limit has not been reduced further cannot be explained, and so there must be other causes also (Examples: improvements in the safety aspects of vehicles, physical segregation of vehicles plying in opposite directions etc.). Among the choices, it is (A) which states this, and is the answer. Both (B) and (C) extend the scope of the information in the passage beyond what can be immediately inferred from it, and are wrong. (D), if true, would have resulted in an increase, and not a decrease, in accidents, and thereby contradicts the given information. (E) contradicts the first part of the given statement that the number of deaths per mile fell abruptly in 1955 as a result of the imposition of the speed limit of 55 miles per hour.
9. It is easy to see that the authors primary concern is the continued existence of his country as a nation. He believes that, in order to ensure its continuing as a nation, active measures must be taken to preserve national security, even though some of these measures may not be morally acceptable. We have been asked to spot that choice which follows logically from the authors statements. Since the author talks only of security measures and not of foreign policy, (A) unnecessarily extends the scope of the information in the passage, and is not a logical conclusion from it. What the author says is that his countrys continued existence as a nation depends upon preserving national security. His thinking therefore applies only to those countries which wish to continue as a nation. (B) extends the scope of the information to cover all countries, and is therefore wrong. The author says that only some active measures that are taken to preserve national security may not be morally acceptable. (C), which implies that all such active measures are immoral, is an uncalled-for generalization of the authors statement, and is wrong. The author says that active steps to preserve the national security must be taken even though some of them may not be morally justified. He does not say that these immoral steps would become moral just because they help the country to preserve itself as a nation. So, (D) also extends the scope of the authors statement beyond what can be immediately inferred, and is not the answer. What the author means by the last sentence is that adherence to morality is less important than ensuring the nations security. (E) states just his, and is the answer. 10. This is a very simple question camouflaged to sound difficult. From the phrase Of women in the primary childbearing age range of eighteen to twentyfour who account for about 40 percent of all births in this country annually ....., we can infer that the remaining about 60% of the annual number of births in the country

New GRE - Logical Reasoning


are accounted for by women who are either less than 18 or older than 24. (B) says just this, and is the answer. (A) looks like the plausible answer, but is incorrect. According to the last sentence of the passage, women in the primary childbearing age range of 18 to 24 account for 40% of all births, and more than 25% of them have no healthcare insurance. This means that only about 10% (which is 25% of 40%) of all mothers do not have healthcare insurance, and the rest of nearly 90% of them do have such insurance. Therefore, (A), which pegs this number at a mere 75%, is wrong. SET 2 Answers 1.A 2.D 3.C 4.C 5.C 6.C 7.B 8.B 9.C 10.B Analysis 1. The given passage means that, irrespective of the skills needed, some jobs traditionally have been paying lower wages while other jobs have traditionally been paying higher wages. It then adds that, for historical reasons, women have generally sought employment in jobs paying lower wages, even if some of these jobs required higher skills. If this is true, the implication is that some jobs which are usually taken up by men pay higher wages than some other jobs which require comparable skills but which have been traditionally taken up by women. It is (A) which states this, and is the answer. None of the other choices follows from the given statement as a necessary corollary. 2. (Marijuana is the name of a habit-forming drug similar to opium and LSD. Syndrome means a group of symptoms which are associated with a particular illness, or physical or mental condition.) The first sentence of the passage mentions the popular assumption that Marijuana causes apathy, depression and loss of motivation in people who use it heavily. The next two sentences go on to dispute this statement and say that, on the contrary, young people who are already depressed, unmotivated and looked down upon by parents, have a tendency to make heavy use of marijuana. The author thus advances the theory that the use of marijuana may not be the cause of depression or amotivation, but may be the result of it. So, it is (D) that would be the most logical next sentence of the passage. Having mentioned the popular belief first, the author says in the second sentence that there is no evidence that the drug does indeed produce the amotivational syndrome. This only shows that he is skeptical of the popular belief, but does not positively rule it out as totally untrue. Even in the next sentence, he only says that young people who are already depressed are more likely to make heavy use of marijuana. Thus, he does not take a positive stand on the question, but merely advances a theory for consideration. So, (A), which uses the word concluded, does not fit into the tenor of the passage, and cannot be the next sentence of the passage. The author does not deny that heavy use of marijuana and amotivation syndrome go together. (His only question is which is the cause and which is the effect.) Therefore (B) is not the answer. (C) also makes an assertive statement which is not warranted by the use of the phrase are more likely than others in the third sentence. (Notice that, in the correct answer (D) which we have already identified, the phrase used is may be a symptom, which is as tentative as the phrase more likely in the previous sentence, and therefore is a logical corollary.) The choice (E) states something which is totally contradictory to the syndrome mentioned in the three previous sentences, and cannot logically follow from them. 3. (Alumni means old students) At present, it is the general council (which consists of all old students) that has the authority to change the constitution. The senate (consisting of 20 members) can decide only routine matters by majority vote. The new proposal seeks to give the power to the senate itself to change the constitution, provided there is unanimous vote in the senate for such change. We have been asked to identify that choice which follows logically from this proposal. The result of the above proposal is that the power to change the constitution, which is now only with the general council, will be transferred to the senate, provided the senate is unanimous about the proposed change. What this implies is that constitutional changes can then be effected without the general councils approval. It is (C) which states this, and is the answer. The proposal itself involves a change in the constitution and, under the rules at present, it will have to be passed by the general council before it can be implemented. (A), which says that the proposal will take effect if it receives the senates unanimous support, is therefore wrong. (B) is wrong, because the narrative says that, currently, the senate decides routine matters by majority vote, and unanimity is not required. So, no individual senator has a veto power at present. The narrative does not say how the members of the senate are selected. So, (D) does not follow from the narrative, and is not the answer. Since the proposal seeks to transfer the power that is now only with the general body of the alumni to the senate, the voice of the alumni in the conduct of university affairs will, in fact, diminish, and not increase. So, (E) is wrong. 4. The given narrative first says that an economy which has a greater division of labour will also need greater coordination. The second sentence gives the reason for this assertion thus: A greater division of labour results in a larger number of specialized producers, leading to a greater number of disruptions of supply and production, thereby necessitating greater coordination. The third sentence then says, There is always more division of labour in market economies than in planned economies. The question then asks us, If the statements above are true, then which of the following must also be true? In other words, we have been asked to identify that choice which necessarily follows logically from the given statements. The third statement is that there is more division of labour in market economies than in planned economies. The first statement is that the greater the division of labour in an economy, the greater the need for coordination. These two statements together imply that there is greater need for coordination in a market economy than in a planned economy. It is choice (C) which states this, and is the answer. (A) contradicts what is stated in the given narrative, because it is market economies that have greater division

New GRE - Logical Reasoning


of labour, and will therefore have more frequent disruptions in supply and production than planned economies. So, (A) is not the answer. Similarly, (B) and (D) also contradict what is stated in the given narrative, and can be discarded. The passage is not on how effectively the division of labour functions in different economies, but is on the quantum of division of labour present in different economies. So, (E) is not relevant to the given conclusion, and is not the answer. 5. The result of the muscular movement mentioned is the expression of smile. What the given statement means is that there is no difference between the smile of blind and deaf children and the smile of normal children. We have been asked to spot the choice which is a logical conclusion that can be drawn from this statement. Children normally learn language as well as patterns of expression by imitation, and this imitation is generally through seeing and hearing the adults. Therefore, if experiments prove that the expression of smile is the same even in children who can neither see nor hear, the conclusion must be that smiling comes naturally to children and is not the result of imitation through sight or sound. So, (C) is the answer. Among the other choices, (A) appears to be a plausible answer. But it is wrong, because elders do not consciously teach gestures to the children, but the children voluntarily pick up these gestures by imitation, and this imitation is normally through sight and sound. Moreover, (A) talks of all facial expressions, whereas the given statement relates to only certain patterns of muscular movement such as those involved in smiling. The other choices are too general, and none of them is an immediate conclusion that can be drawn from a study of children who are both deaf and dumb. 6. The simple statement made by Joe is that all Americans like ice-cream. James controverts him saying No, that is not true, and seeks to substantiate his rebuttal by saying that he knows a couple of Americans who just love chocolates. If James had misunderstood Joe as saying Only Americans like ice-cream, his response would not have been I know a couple of Americans who just love chocolates. So, (A) is not the answer. (If, on the contrary, James response was, I know a couple of Europeans who just love ice-cream, (A) would have been the answer.) If James had misunderstood Joe as saying I do not like ice-cream, his response would not have been I know a couple of Americans who just love chocolates. So, (B) is not the answer. If he had misunderstood Joe as saying, Americans like only ice-cream (and nothing else), his counterstatement, I know a couple of Americans who just love chocolates (and therefore what you say is not true) would have logical force. So, (C) is the answer. Obviously, (D) is not related to what either Joe or James says. Since Joe does not mention chocolates at all, the question of James misunderstanding him to mean that those who like ice-creams will not like chocolates does not arise. So, (E) is not the answer. 7. The given statement makes a positive assertion that any car built after 1965 must have harness-style seat belts in the front seats. The question asks you to examine the five choices given below and pick out the one from which the given statement can logically follow. (A) states that harness-style seat belts first appeared in cars in the late 1950s. From this statement alone, we cannot logically infer that all cars built after 1965 must have such belts. So, (A) has to be discarded. The second part of (B) specifically lays down that all cars built after 1965 were required to have seat belts in the front seats, and the first part says that no seat belts other than harness-style ones were used in front seats of cars after 1965. By combining these together, it can be concluded that if a car was built after 1965, it has harness-type seat belts in the front seats. So, (B) is the answer. (C) states that some cars built before 1965 have harness-style seat belts in the front seats. The only conclusion that can follow from this statement is that many cars built before 1965 did not have harness-style belts in the front seats. You cannot conclude from this statement that all cars built after 1965 had such belts in the front seat. So, (C) is not the answer. (D) talks generally of seat belts, and does not even mention harness-type seat belts. Secondly, it only implies that many people who had seen the usefulness of seats belts in racing cars had them installed in their cars voluntarily. So, we cannot draw a conclusion from (D) that all cars built after 1965 definitely have seat-belts, and that too of the harness-type. So, (D) is not the answer. (E) can only lead us to infer that, if a car was built after 1965, it will have some type of seat belts in the front seats. It does not follow that the seat belts will be of harness-type only. So, (E) is not the answer. 8. We learn from the given information that (i) all empty canvasses sold by retailers in USA and Europe to artists were made in Europe; (ii) some of these canvasses bear the names and addresses of the retailer, either European or American, selling it to an artist; (ii) the wooden stretchers on which the art by a US artist was mounted was always made of American wood; (iii) finished paintings imported into USA from Europe were always mounted on European stretchers, though these were often replaced by American stretchers after import. In the case of (A), it is not probable that a European retailer would have sold an empty canvas to an American painter. It is more logical to surmise that the painting was done by a European artist and the stretcher was changed to American wood after its import to USA. So, (A) is not the answer. (B) is likely to be true because the American retailer is not likely to have sold it to a European painter in those pre-industrial days when trans-Atlantic travel by artists would have been improbable. So, (B) is the answer In the case of (C), it is not likely that an American retailer would have sold a canvas to a European artist. Even if a European artist had bought one from him, his finished painting would have arrived in USA mounted on a European stretcher , though, in some cases, these stretchers might have been replaced by American stretchers after arrival. Consequently, no European painting is likely to be found without a stretcher. So, (C) is not the answer. (D) relates to a canvas that is unmarked, and might therefore have been sold either by a European retailer to a European painter or by an American retailer to an American painter. The painting also might be either by a European or an American artist, because even European

10

New GRE - Logical Reasoning


paintings were often remounted on American stretchers. So, (D) is not a greater probability than (B). (E) is also not the answer, because it is given that all European paintings arrived in USA on a stretcher and, often, were remounted on American stretchers after arrival. So, a painting without a stretcher is more likely to be an American painting than a European painting. 9. The first sentence calls the outlawing by a local government of all strikes by its workers as a costly mistake, and gives the reason as because all its labor disputes must then be settled by binding arbitration. This implies that a recourse to binding arbitration always results in less advantageous results for the local government than it does for the workers. This is also confirmed by the statement that, in such cases, the arbitrators have to decide without the benefit of guidance from any negotiated public-sector labor settlements, meaning that such negotiated settlements would be more advantageous to the local government than the arbitrators awards. It is (C) which says this, and is the answer. None of the other choices follows logically from the given information. 10. The given statement is whenever it rains, the streets and sidewalks are both wet. It does not follow that the converse (namely that whenever the streets and sidewalks are wet, it is raining) is true, because the wetness of the streets and sidewalks might have been caused by reasons other than rain - such as operation of the fire service or cleaning by municipal staff or leakage of water from a water truck etc. So (A) and (C) need not be true. If it is raining, then both the streets and the sidewalks must be wet. If the streets alone are wet but the sidewalks are not wet, the wetness of the streets must be due to some other reason (such as the operation of the fire service, cleaning by the municipal staff, or leakage of water from a water truck etc.), and not due to rain. So, (B) must be true. So, you must choose only (B) as the answer.

11

Potrebbero piacerti anche